feat(geometria): aggiorna gli appunti del 24/03/2023 (e anche della lezione precedente ancora)

main
parent 4065e6bd09
commit 0365fcce4c

@ -70,7 +70,7 @@ Pertanto $\varphi_A(x) = \mcm(\varphi_{A,\Vec{e_1}}(x),\varphi_{A,\Vec{e_2}}(x),
di $\varphi_f$ e lo divide, deve anche valere che $\varphi_{f,\Vec{v}} = \varphi_f$. \\ di $\varphi_f$ e lo divide, deve anche valere che $\varphi_{f,\Vec{v}} = \varphi_f$. \\
\li Nella base ordinata $\basis$ costituita dai primi $n$ vettori del ciclo di $\Vec{v}$, la matrice associata di $f$ è della forma: \li Nella base ordinata $\basis$ costituita dai primi $n$ vettori del ciclo di $\Vec{v}$, la matrice associata di $f$ è della forma:
\[ M_{\basis}(f) = \begin{pmatrix} \[ M_{\basis}(f) = C_{\varphi_f} = \begin{pmatrix}
0 & 0 & \ldots & \ldots & 0 & -a_0 \\ 0 & 0 & \ldots & \ldots & 0 & -a_0 \\
1 & 0 & \ldots & \ldots & 0 & -a_1 \\ 1 & 0 & \ldots & \ldots & 0 & -a_1 \\
0 & 1 & \ddots & & \vdots & \vdots \\ 0 & 1 & \ddots & & \vdots & \vdots \\
@ -79,6 +79,18 @@ Pertanto $\varphi_A(x) = \mcm(\varphi_{A,\Vec{e_1}}(x),\varphi_{A,\Vec{e_2}}(x),
\end{pmatrix}, \] \end{pmatrix}, \]
dove gli $a_i$ sono i coefficienti di $\varphi_f(x) = \varphi_{f,\Vec{v}} = x^n + a_{n-1}x^{n-1} + \ldots + a_1 x + a_0$. dove gli $a_i$ sono i coefficienti di $\varphi_f(x) = \varphi_{f,\Vec{v}} = x^n + a_{n-1}x^{n-1} + \ldots + a_1 x + a_0$.
Tale matrice è detta \textbf{matrice compagna} del polinomio $\varphi_f$.
\li Ogni polinomio $q \in \KK[x]$ è il polinomio caratteristico, a meno del segno, della propria matrice compagna. In particolare $p_{C_q}(\lambda) = (-1)^n q(\lambda)$, dove $n := \deg q$. Infatti, se $n = 0$, $C_q = (-a_0) \implies p_{C_q}(\lambda)
= -\lambda -a_0 = -(\lambda + a_0)$. Altrimenti, assumendo che la tesi sia vera per $i \leq n$, si osservi che:
\[ p_{C_q}(\lambda) = (-1)^n a_0 -\lambda p_{C_{q'}}(\lambda), \qquad q'(\lambda) = \frac{q(\lambda) - a_0}{\lambda}, \]
ossia, dacché $\deg q' = n - 1 < n \implies p_{C_{q'}}(\lambda) = (-1)^{n-1} q'(\lambda)$,
\[ p_{C_q}(\lambda) = (-1)^n a_0 - \lambda (-1)^{n-1} q'(\lambda) = (-1)^n a_0 - \lambda (-1)^{n-1} \frac{q(\lambda) - a_0}{\lambda} = q(\lambda). \]
\li Inoltre, osservando che $\basis = (\e1, C_q \e1 = \e2, C_q^2 \e1 = \e3, ..., C_q^{n-1} \e1 = \e n)$ è esattamente
la base canonica di $\KK^n$, essendo $\basis$ una base ciclica di $C_q$ su $\KK^n$ deve valere che $\varphi_{C_q}$
ha grado $n$, e quindi che $p_{C_q} = \pm \varphi_{C_q}$. Si conclude allora che $\varphi_{C_q} = q$.
\end{remark} \end{remark}
\begin{proposition} \begin{proposition}

@ -11,16 +11,64 @@
\maketitle \maketitle
\begin{center} \begin{center}
\Large \textbf{Es. 24/03/2023} \Large \textbf{Esercitazione: la forma canonica di Jordan e gli autospazi generalizzati}
\end{center} \end{center}
%TODO: aggiungere matrice compagna di q(x) negli scorsi appunti di es. e calcolare polinomio minimo e caratteristico. \begin{note}
Nel corso del documento, per $f$ si intenderà un generico endomorfismo di $\End(V)$, e per $V$
verrà inteso uno spazio vettoriale di dimensione finita $n$ su un campo $\KK$ algebricamente
chiuso, qualora non specificato diversamente.
\end{note}
%TODO: recuperare la lezione e approfondirla. Sia $f \in \End(V)$. Si osservino allora le seguenti catene ascendenti:
\begin{gather}
\{\vec0\} \subsetneq \Ker f \subsetneq \Ker f^2 \subsetneq \cdots \subsetneq \Ker f^{k-1} \subsetneq \Ker f^k = \Ker f^{k+1} = \cdots, \\
\{\vec0\} \subsetneq \Im f \subsetneq \Im f^2 \subsetneq \cdots \subsetneq \Im f^{k-1} \subsetneq \Im f^k = \Im f^{k+1} = \cdots,
\end{gather}
\begin{remark} Sia la $(1)$ che la $(2)$ devono stabilizzarsi allo stesso $k \in \NN$, per la cosiddetta decomposizione di Fitting.
Sia $k$ l'indice della decomposizione di Fitting su $f$. Allora Sempre per tale decomposizione vale in particolare che:
vale che $k = \mu_a(0)$ in $\varphi_f$.
\[ V = \Ker f^k \oplus \Im f^k. \]
\begin{remark} Si possono fare alcune osservazioni riguardo la decomposizione di Fitting. \\
\li Sia $\Ker f^k$ che $\Im f^k$ sono $f$-invarianti: $\vec v \in \Ker f^k \implies f^k(f(\vec v)) = f(f^k(\vec v)) = \vec0 \implies f(\vec v) \in \Ker f^k$ e $\vec v \in \Im f^k \implies \vec v = f^k(\vec w)$, $f(\vec v) = f(f^k(\vec w)) = f^k(f(\vec w)) \in \Im f^k$. \\
\li $\restr{f}{\Ker f^k}$ è nilpotente: $(\restr{f}{\Ker f^k})^k = \restr{f^k}{\Ker f^k} = 0$. \\
\li $\restr{f}{\Im f^k}$ è invertibile: $\Ker \restr{f}{\Im f^k} = \Ker f \cap \Im f^k \subseteq \Ker f^k \cap \Im f^k = \{\vec 0\}$, e quindi $\restr{f}{\Im f^k}$ è iniettiva; quindi $\restr{f}{\Im f^k}$ è anche invertibile, essendo un endomorfismo. \\
\li Poiché $\restr{f}{\Ker f^k}$ è nilpotente, $p_{\restr{f}{\Ker f^k}}(\lambda) = \lambda^d$, dove
$d = \dim \Ker f^k$. Inoltre
$\varphi_{\restr{f}{\Ker f^k}}(\lambda) = \lambda^k$: se infatti $\varphi_{\restr{f}{\Ker f^k}}(\lambda) = \lambda^t$
con $t < k$, varrebbe sicuramente che ${\restr{f}{\Ker f^k}}^{k-1} = \restr{f^{k-1}}{\Ker f^k} = 0$, ossia che
$\Ker f^k \subseteq \Ker f^{k-1}$, violando la minimalità di $k$, \Lightning. \\
\li Dal momento che vale la decomposizione di Fitting e che $\varphi_{\restr{f}{\Ker f^k}}$ e $\varphi_{\restr{f}{\Im f^k}}$ sono coprimi tra loro (il primo è diviso solo da $t$, mentre il secondo non è diviso da $t$), $\varphi_f = \mcm(\varphi_{\restr{f}{\Ker f^k}}, \varphi_{\restr{f}{\Im f^k}}) = \varphi_{\restr{f}{\Ker f^k}} \varphi_{\restr{f}{\Im f^k}}$. Si conclude quindi che $k = \mu'_a(0)$ rispetto a $\varphi_f$, ossia la molteplicità algebrica di $0$ in
tale polinomio. Analogamente si osserva che $t = \mu_a(0)$ rispetto a $p_f$, ossia la molteplicità algebrica
dell'autovalore $0$ in $f$, e quindi che $\mu_a(0) \geq k$.
\end{remark} \end{remark}
Reiterando la decomposizione di Fitting (o applicando il teorema di decomposizione primaria), si ottiene
infine la seguente decomposizione di $V$:
\[ V = \Ker (f - \lambda_1 \Id)^{\mu_a(\lambda_1)} \oplus \cdots \oplus \Ker (f - \lambda_m \Id)^{\mu_a(\lambda_m)}, \]
dove $m$ è il numero di autovalori di $V$. Si può riscrivere questa identità ponendo $n_i := \mu'_a(\lambda_i)$ in
$\varphi_f$:
\[ V = \Ker (f - \lambda_1 \Id)^{n_1} \oplus \cdots \oplus \Ker (f - \lambda_m \Id)^{n_m}. \]
Si deduce da questa identità che $f$ è diagonalizzabile se e solo se $n_i = 1$ $\forall i \leq m$.
%TODO: aggiungere come proposizione e approfondire
\begin{exercise}
Sia $A \in M(n, \CC)$ invertibile. Dimostrare allora che se $A^3$ è diagonalizzabile, anche $A$ lo è.
\end{exercise}
\begin{solution}
Se $A^3$ è diagonalizzabile, per la precedente osservazione, $\varphi_{A^3}(t) = \prod_{i=1}^m (t - \lambda_i)$,
dove $m$ è il numero di autovalori distinti di $A^3$. Allora, detto $p(t) = \prod_{i=1}^m (t^3 - \lambda_i)$, vale che
$p(A) = 0$, ossia che $\varphi_A \mid p$. Dal momento che $A$ è invertibile, anche $A^3$ lo è, e quindi
$\lambda_i \neq 0$ $\forall i \leq m$. Poiché $p$ è allora fattorizzato in soli termini lineari distinti,
anche $\varphi_A$ deve esserlo, e quindi $A$ deve essere diagonalizzabile.
\end{solution}
\end{document} \end{document}

@ -78,6 +78,12 @@
\newcommand{\RRbar}{\overline{\RR}} \newcommand{\RRbar}{\overline{\RR}}
% Spesso utilizzati al corso di Geometria 1. % Spesso utilizzati al corso di Geometria 1.
\let\Im\undefined
\DeclareMathOperator{\Im}{Im}
\newcommand{\restr}[2]{
#1\arrowvert_{#2}
}
\DeclareMathOperator{\val}{val} \DeclareMathOperator{\val}{val}
\DeclareMathOperator{\Span}{Span} \DeclareMathOperator{\Span}{Span}
\newcommand{\charpoly}[1]{p_{#1}} \newcommand{\charpoly}[1]{p_{#1}}

Loading…
Cancel
Save